Determine whether the first system of equations is equivalent to the second system of equations. Explain.

3x + 5y = 1

18x + 30y = 6

2x-6y=38 10x - 30y = 190

Answers

Answer 1

Answer:

1 Answer:  Since these two lines are the same line, they have infinite solutions Answer: So first thing you gotta do is multiply the first equation by 5. Your first equation will be. Since these two lines are the same line, they have infinite solutions.

Another Answer: You would have to multiply either equation in order to get one of the variables to cancel.

I would multiply the first equation by -5.

-5(3x + 6y = -2)

-15x -30y = 10

15x + 30y = -10

0 + 0 = 0

Since everything cancels and the equation is true, the solution is Infinitely many solutions, or all real numbers.

Step-by-step explanation:


Related Questions

Rationalise I) 6/√5+√2
ii) 1/8+5√2​

Answers

Given :

6/√5+21/8+5√2

To find :-

The denominator in rationalized form .

Solution :-

Using Identity :-

a² - b² = (a+b)(a-b)

Solution 1:-

6/√5+26(√5-2)/(√5+2)(√5-2)6(√5-2)/√5²-2²6(√5-2)/5-465 - 12

Solution 2:-

1/8+5√21(8-5√2)/(8+5√2)(8-5√2)8-5√2/8²-(5√2)²8-5√2/64-508-52/14

find th slope of the line that passes through each pair y2 of points E(4, 0), F(5, 5​

Answers

Step-by-step explanation:

the slope = (5-0)/(5-4)

= 5/1

= 5

Answer:

b

Step-by-step explanation:

hola so today i have just showed you my answer i got and it was right let me know if you got it wrong>

3 3/4 ÷ 5 5/8
write in simplest form. Check by multiplying.
pls teach me how to answer it. I'll give brainliesnt if correct.​

Answers

Step-by-step explanation:

3¾ divided by 5⅝

4*3+3 = 15/4

8*5+5 = 45/8

15/4 * 8/45 =

2/3

Compare & Contrast: What is different about complementary and supplementary angles? What is the same?​

Answers

Complementary: When two angles add up to 90 degrees

Supplementary: When two angles add up to 180 degrees

Similarities are both involve two angles, the difference is the total degrees they add up to.

What is the domain and the range to this

Answers

Answer:

Look Down below :P

Step-by-step explanation:

If you were looking for if it even has a domain and range let's say if the point (15/4,7) should have read (15/4,17) then a plot of these points appears to show a hyperbola and the x and y (f(x)) axes look like Asymptotes. If the general function has the form f(x)=a/x, then there should be a consistent value for a which can be found be substituting the points in the equation. Take one point (17,15/4) and we get 15/4=a/17, from which a=255/4. f(x)=255     x/4. We have no negative values as examples so we cannot assume that the function is defined for negative values of x. f(x)=255 x/4 fits all the points except for (15/4,7), which is possibly mis-typed. The domain of the function is therefore all values of x>0 and the range of f(x) is f(x)>0, assuming continuity and extrapolation of the function.

If the point (15/4,7) is valid then f(x) as defined above is invalid or incomplete. f(x) may be a piece wise function where f(x)=7 if and only if x=15/4. For 0<x<15/4 and x>15/4, f(x)=255 x/4. These would be the domain for x. The range of f(x) is f(x)>0 and f(x)=7 only when x=15/4.

3) (2.01) Identify the domain and range. Then, determine whether the
relation is a function or not. Explain your reasoning.
Relation: (-1,0) (2, 4) (0, 3) (2, 1) (6,5)
Domain:
Range:
Is this relation a function? Why or why not?

Answers

Domain: {-1,2,0,6}
Range: {0,4,3,1,5}
The relation is not a function because the domain 2 has more than one range, 1 and 4.

Please answer this question!!

Answers

Answer:

the x- coordinate of the turning point is - [tex]\frac{5}{2}[/tex]

Step-by-step explanation:

Given

y = x² + bx + c with y- intercept = (0, - 14 ) then c = - 14 , so

y = x² + bx - 14

To find b substitute (2, 0 ) into the equation

0 = 2² + 2b - 14

0 = 4 + 2b - 14

0 = - 10 + 2b ( add 10 to both sides )

10 = 2b ( divide both sides by 2 )

5 = b , then

y = x² + 5x - 14

Given a parabola in standard form y = ax² + bx + c ( a ≠ 0 )

Then the x- coordinate of the turning point is

x = - [tex]\frac{b}{2a}[/tex]

y = x² + 5x - 14 ← is in standard form

with a = 1 and b = 5 , then

x- coordinate of turning point = - [tex]\frac{5}{2}[/tex]

It must be -5/2 I guess

Write an equation of the line (4,3) and (9,-2)

Answers

Answer:

y= -x + 7

Step-by-step explanation:

to get the equation of a line is y=mx+b

the slope, m, would be calculated (y1-y2)/(x1-x2)

thus we would substitute in (3--2)/(4-9)

which simplifies to 5/-5

or -1

now we know that m= -1

to solve for b we have to put the equation into point-slope form which is

y-y1= m(x-x1)

this would be y-3 = -(x-4)

y-3 = -x+4

y= -x+7

Rewrite the quadratic equation in standard form 4x^2=2x-6

Answers

Answer:

Answer Below....

Step-by-step explanation:

Answer:

4x^2 - 2x + 6

Step-by-step explanation:

The "standard form" of the quadratic equation is ax^2 + bx + c.

Here we have 4x^2 = 2x - 6, whose terms must be rearranged to resemble ax^2 + bx + c.  Term 4x^2 stays where it is, but + 2x - 6 on the right side must be moved to the left side, which results in:  4x^2 - 2x + 6

Multiply what time 1/2 to get 540 as answer

Answers

The Answer is 1080! Hope this helps,
Multiplying by 1/2 is like dividing by 2, so what divided by 2 is 540. This can help you figure out the answer.

Answer:

1080

Step-by-step explanation:


A store owner sold 4 paint brushes for $12.00 and 8 pens for $3 20. What is the difference between the unit price of a paint brush and the unit price of a pen?

Answers

One of the units of money and the o her is p of b

You are shopping for school supplies. You want to buy 8 notebooks for $1.50 each. Show how you can use the distributive property to find the total cost of the notebooks mentally.
8x $150=8(1+ )
=8(1)+8( )
=8+ ( )
=$ ( )

Answers

Answer:

12

Step-by-step explanation:

8x1+8x0.5

8+4

12

12$ is the total cost

please help me with this.....​

Answers

Answer:

answer below

Step-by-step explanation:

C. i) y = 23,000(1 - 0.15)^t

ii) y = 23,000(1 - 0.15)^t

y = 23,000 x 0.85^4

y = 23,000 x 0.52200625

y = $12,006.14

y = $12,006. when rounded to the nearest dollar

This is what I got

Each marble bag sold by Chris's marbles company contains 7 red marbles for every 5 orange marbles , if a bag had 30 orange marbles how many red marbles does it contain

Answers

If a bag has 30 orange marbles then that means it has 42 red marbles

Step-by-step explanation:

If there are 7 red marbles for every 5 orange marbles then what you would do is.

30/5=6

6*7=42

Each marble bag sold by Chris's marbles company contains 7 red marbles for every 5 orange marbles , if a bag had 30 orange marbles Then the bag contains 42 red marbles.

In the problem, we are given that each bag of marbles contains 7 red marbles for every 5 orange marbles. We need to find the number of red marbles when there are 30 orange marbles in the bag.

Let's represent the number of red marbles as R and the number of orange marbles as O.

According to the information provided, we have the following relationship:

R / O = 7 / 5

Now, we are given that the number of orange marbles (O) in the bag is 30. We can substitute this value into the equation:

R / 30 = 7 / 5

To find the number of red marbles (R), we can cross-multiple

R = (7 / 5) * 30

R = 42

Therefore, the bag contains 42 red marbles.

In summary, we used the given ratio of red to orange marbles in the bag to set up a proportion and then solved for the number of red marbles when the number of orange marbles was known. This allowed us to find that there are 42 red marbles in the bag.

To know more about Equation here

https://brainly.com/question/1905286

#SPJ2

Juan pays $500 per month in rent, a semiannual car insurance premium of $800, and an annual health club membership fee of $900.

Answers

Answer:

400 is the answer

Step-by-step explanation:

900-800=100

500-100=400

which expression is equivalent to the ghen expression?

Answers

Answer:

3√5

Step-by-step explanation:

Square root of 45

Square Root of 45: √45 = 6.708

We can find the square root of 45 using the following steps:

Step 1: Check whether the number is a perfect square or not. In this case, 45 is not a perfect square as it cannot be broken down into a product of two same numbers.

Step 2: Once the number is checked, the following process needs to be followed:

If the number is a perfect square, it can be written in this format √x2 = x

If it is not a perfect square, the square root is found using the long division method. It can also be written in its simplified radical form.

Since 45 is not a perfect square, we find its square root using the long division method. The simplified radical form of the square root of 45 is given below.

Simplified radical form of the square root of 45-

45 can be written as a product 9 and 5. Hence, square root of 45 can be expressed as, √45 = √(9 × 5) =  3√5. 45 is not a perfect square, hence, it remains within roots. The simplified radical form of the square root of 45 is 3√5.

---------------------------------------------------------------------------------------------------------------

9514 1404 393

Answer:

  C.  3√5

Step-by-step explanation:

Your calculator can help you choose the correct answer.

  [tex]\sqrt{45}=\sqrt{9\cdot5}=\sqrt{9}\cdot\sqrt{5}=\boxed{3\sqrt{5}}[/tex]

_____

Compare the decimal values using your calculator:

  √45 ≈ 6.7

  5√3 ≈ 8.7

  9√5 ≈ 20.1

  3√5 ≈ 6.7

  5√9 = 15

Help me with this pleaseee

Answers

Answer:

option c

Step-by-step explanation:

Volume = Length * Width * Height

this is a pretty easy problem cuz all u need to do is find which one of the options has a product of 729.

option a) 7 * 8 * 12

well 7 * 8 * 12 equals 672, so that cannot be the box

option b) 8*9*11

8*9*11 = 792. cant be the box cuz 792 does not equal 729

option c) 9*9*9

9*9*9=729

option c is the box cuz 729 = 729

636 to the nearest one hundred

Answers

Answer:

600

Step-by-step explanation:

It is less than 650

Answer:

600

Step-by-step explanation:

Becuase when you round by 100's you have to see if the number is greater than 50 or not. In this case it is 636 and it is not greater, then 650.

Is y=-x^3+2x linear or non linear

Answers

Answer:

It's non=linear

Nannamakamakakakaannanana

need a lil help please

Answers

Answer:

B(2, 4)

Step-by-step explanation:

Given points:

A(-1, -9) and M(0.5, -2.5)

Let the coordinates of B are (x, y)

Use midpoint formula to determine the point B:

0.5 = (- 1 + x)/2 ⇒ 1 = -1 + x ⇒ x = 1 + 1 = 2-2.5 = (-9 + y)/2 ⇒ -5 = -9 + y ⇒ y = -5 + 9 = 4

pls pls pls pls help me im literally dying help its due today pls help pls im gonna die pls

Answers

Answer:

What?

Step-by-step explanation:

Help with what?

Answer:

whats the question?

Step-by-step explanation:

Given the associated quantity demanded, Q1, and quantity supplied, Q2, which of the following describes the quantity Q2−Q1?

A.the surplus resulting from the price floor Pc
B. the shortage resulting from the price floor Pc
C.the surplus resulting from the price ceiling Pc
D. the shortage resulting from the price ceiling Pc

Answers

Answer:

1.Maximum price below the market equilibrium price

2.Producers from low market prices

3.Lead to a shortage as prices are kept from rising to their equilibrium level

4.A price ceiling of $1,000

5.The surplus resulting from the price floor Pc

Step-by-step explanation:

I just took the Price Ceilings and Floors Quick Check

Have a great day/night!

simplify x ^2 ⋅ x ^−12

Answers

Answer:

x^2*x^-12= 1/x^10

Which graph shows exponential growth?
5
4
3. 2
1
2
54 -3 -2 -1
1 2 3 4 5 X
-2
-3
-4
-5
ly I

Answers

Answer:

54 -3 -2 -1

Step-by-step explanation:

I hope this help!!!

you must use substitution method
x+y=7
2x+y=10

Answers

Answer:

x = 3, y = 4

Step-by-step explanation:

First, set the first equation equal to x. Note the equal sign, what you do to one side, you do to the other.

Subtract y from both sides of the equation:

x + y = 7

x + y (-y) = 7 (-y)

x = 7 - y

Next, plug in 7 - y for x in the second equation:

2x + y = 10

2(7 - y) + y = 10

Simplify. First, distribute 2 to all terms within the parenthesis:

2(7 - y) = (2 * 7) + (2 * -y) = 14 - 2y

Next, combine like terms. Like terms are terms with the same amount of the same variables:

14 - 2y + y = 10

14 (-2y + y) = 10

14 - y = 10

Isolate the variable, y. First, subtract 14 from both sides of the equation:

-y + 14 (-14) = 10 (-14)

-y = 10 - 14

-y = -4

Next, isolate the variable, y, by dividing -1 from both sides of the equation:

(-y)/-1 = (-4)/-1

y = -4/-1 = 4

Next, plug in 4 for y in one fo the given equations:

x + y = 7

x + (4) = 7

Isolate the variable, x. Subtract 4 from both sides of the equation:

x + 4 (-4) = 7 (-4)

x = 7 - 4

x = 3

x = 3, y = 4

If 461z5 is a multiple of 9, then the least possible value of z​

Answers

Answer:

46125

z = 2

Step-by-step explanation:

46125 / 9 = 5125

Which function matches the graph below?

Answers

Try number 1 hope it’s correct

How can variables and mayhem expressions be used to show different ways of a pattern?

Answers

Answer:

A variable is a letter or symbol, such as a, b, or x, that represents a number. An algebraic expression is a combination of one or more variables; it may include numbers and operation signs. For example, 2 d 5 is an algebraic expression that could represent two times the figure number plus five.

Step-by-step explanation:

Hope it helps u..pls mark me the Brainliest.pls...

What is the image of the point (-5, -9) after a rotation of 90° counterclockwise
about the origin?

Answers

Answer:

(9, -5)

Step-by-step explanation:

If you rotate a figure 90 degrees counterclockwise, the coordinates will go from (x, y) to (-y, x).

So, let's start out with -y. The y-value of (-5, -9) is -9, and the negative value of that is -(-9) or 9. That goes first in our point: (9, x.) Now, we just keep our x-value the way it is, -5, and put that in our y-value: (9, -5). And there's our point after a 90 degree counterclockwise rotation.

(9,-5) is write for that equation

Four circles, each with a radius of 2 inches, are
removed from a square. What is the remaining area of
the square?
(16 – 4pi) in.2
(16 – pi) in.2
(64 – 16pi) in.2
(64 – 4pi) in.?

Answers

Area of each circle=π(2)^2=4πin^2

Area of 4circles=4(4π}+16π in^2

Hence the area if square= 4^3=64

Remaining area

[tex]\\ \sf\longmapsto (64-16\pi)in^2[/tex]

Other Questions
stan loona SLAY BOMB yass bomb stan loona How would you expect the rate of an enzyme-controlled reaction to change if the temperature was raised from 35 C to 55 C? Explain your answer. The owner of a large restaurant is considering a new no tipping policy and wants to survey a sample of employees. The policy would add 20% to the cost of food and beverages and the additional revenue would be distributed equally among servers and kitchen staff. please help this is trigonometry 10+10 what is the answer what is the underlying force that drives plate tectonics? !!!!HELP!!!! WILL MARK BRAINLYEST The table contains the points of an absolute value function. Compare it to the function f(x)=2|x7|4 .x g(x)1 22 13 24 3Which function has the largest maximum and what is the value of the largest maximum? A. The function g(x) has the largest maximum which is 1 .B. The function g(x) has the largest maximum which is 4 .C. The function f(x) has the largest maximum which is 4 .D. The function f(x) has the largest maximum which is 1 . Which is a potential counterclaim to this claim?Claim: People should use cloth napkins instead ofpaper napkins because cloth napkins are better for theenvironmentThousands of trees are cut down each year to makepaper napkins.Using recycled paper napkins is better for theenvironment than using white napkinsCloth napkins can also be harmful to theenvironment because washing them wastes waterand energyWhen paper napkins are bleached white, dangerouschemicals get into our water supply which molecule do indole positive bacteria cleave to form indole Community Policing does which of the following? *A.Facilitates positive interactions between community members and the policeB.Can promote fair and impartial policingC.Both A and BD.Neither A and BOther: When titrating a NaF(aq) solution with HCl(aq), the equivalence point will occur at a pH value ______ .a) above 7 because it is determined by NaF(aq)b) below 7 because HCl(aq) is a strong acidc) below 7 because it is determined by HF(aq)d) at 7 because it is determined by H2O(l) 4.what equation passes through the 2 points (2, 1) and (6, 9) Help ma pls ! As the sun goes down the air contains a certain amount of water vapor in the air then begins to cool and eventually the air reaches the dew point.What is the dew point and what form at this point?please no bs ill mark the crown thingy thingy Solve the system by graphing.1-3x - y = -10[4x 4y = 8 please help i cant fail this text -2 of what equals to -6 Brody's fishing cooler is shaped like a rectangular prism. The inside of the cooler is 10 inches wide, 18 inches long, and 6 inches tall. What is the volume of the cooler? 5.2500000000 -2 1/67 What happens if you run 100 laps in a day? Select the letter of the correct answer.The Article primarily discussesA. some of the unwritten rules that most early jazzmusicians followed, including playing as an ensembleB. how Louis Armstrong revolutionized music with hislegendary trumpet solos and distinctive voiceC. some of the cover songs that today's singers performby singing someone else's song in a way that fits theirown styleD. how Louis Armstrong criticized U.S. President DwightD. Eisenhower's handling of school desegregation